1
$\begingroup$

In his article "Polynomials with Integer values" (Resonance), Sury proved an interesting lemma:

If $P$ is a non constant, integral valued, polynomial, then the number of prime divisors of his values $\{P(m)\}_{m\in \mathbb Z}\ $ is infinite, i.e. not all the terms of the sequence $P(0),\ P(1),\ P(2)\ldots $ can be build from finitely many primes.

Does anyone know from where this theorem is taken, or is it a discovery of Sury?

Sil
18.2k3 gold badges45 silver badges85 bronze badges
asked Oct 24, 2021 at 18:09
$\endgroup$
4
  • 2
    $\begingroup$ The set of numbers that are divisible by a given finite set of primes thins out rather quickly; it forces the values of $P$ to grow exponentially. The idea is simple but the details are a bit cumbersome. $\endgroup$ Commented Oct 24, 2021 at 21:04
  • $\begingroup$ @Servaes. Do you mean you have a proof different from that of Sury? $\endgroup$ Commented Oct 25, 2021 at 12:30
  • $\begingroup$ I don't know what Sury's proof is, so I don't know whether my proof is different. $\endgroup$ Commented Oct 25, 2021 at 17:47
  • $\begingroup$ So, just give the idea of your proof in an answer! (in a form a bit more elaborated than you comment) $\endgroup$ Commented Oct 25, 2021 at 18:10

1 Answer 1

2
$\begingroup$

As per my information, Schur proved this result for the first time. I also gave an alternative proof to this result which you may find here

The reference of Schur's work can be found in the reference section of the paper.

Thanks.

answered Mar 30, 2023 at 8:41
$\endgroup$
4
  • $\begingroup$ The link to your paper don't work. $\endgroup$ Commented Mar 30, 2023 at 8:52
  • $\begingroup$ sorry,check now. $\endgroup$ Commented Mar 30, 2023 at 9:12
  • $\begingroup$ Thank you. You may be interested in my own article, which is quite elementary too: arxiv.org/pdf/2112.14711.pdf $\endgroup$ Commented Mar 30, 2023 at 9:16
  • $\begingroup$ I read the article. You could use the standard notation Int(R) for the ring of integrally valued polynomial. Thanks. $\endgroup$ Commented Mar 30, 2023 at 9:27

You must log in to answer this question.

Start asking to get answers

Find the answer to your question by asking.

Ask question

Explore related questions

See similar questions with these tags.